Difference between revisions of "2004 AMC 12A Problems/Problem 6"

(See Also)
Line 6: Line 6:
 
==Solution==
 
==Solution==
 
<cmath>\begin{eqnarray*}
 
<cmath>\begin{eqnarray*}
U-V&=&2004^{2005}\\
+
U-V&=&2004*2004^{2004}\\
 
V-W&=&2004^{2004}\\
 
V-W&=&2004^{2004}\\
 
W-X&=&2001*2004^{2004}\\
 
W-X&=&2001*2004^{2004}\\

Revision as of 15:49, 12 July 2017

Problem

Let $U=2\cdot 2004^{2005}$, $V=2004^{2005}$, $W=2003\cdot 2004^{2004}$, $X=2\cdot 2004^{2004}$, $Y=2004^{2004}$ and $Z=2004^{2003}$. Which of the following is the largest?

$\mathrm {(A)} U-V \qquad \mathrm {(B)} V-W \qquad \mathrm {(C)} W-X \qquad \mathrm {(D)} X-Y \qquad \mathrm {(E)} Y-Z \qquad$

Solution

\begin{eqnarray*} U-V&=&2004*2004^{2004}\\ V-W&=&2004^{2004}\\ W-X&=&2001*2004^{2004}\\ X-Y&=&2004^{2004}\\ Y-Z&=&2003*2004^{2003} \end{eqnarray*}

After comparison, $U-V$ is the largest. $\mathrm {(A)}$

See Also

2004 AMC 12A (ProblemsAnswer KeyResources)
Preceded by
Problem 5
Followed by
Problem 7
1 2 3 4 5 6 7 8 9 10 11 12 13 14 15 16 17 18 19 20 21 22 23 24 25
All AMC 12 Problems and Solutions

The problems on this page are copyrighted by the Mathematical Association of America's American Mathematics Competitions. AMC logo.png